Answer (B) is correct . The expected sales levels should be weighted by the individual probabilities of their occurrence. The midpoint of each sales level is used as the estimate ? for that level. Thus, sales are expected to be 380 units. 100 ¡Á 15% = 15 300 ¡Á 45% = 135 500 ¡Á 25% = 125 700 ¡Á 15% = 105 380 Answer (A) is incorrect because The figure of 480 is based on the maximum value in each range. Answer (C) is incorrect because The maximum value in the modal range is 400. Answer (D) is incorrect because The highest estimate is 800.
|